NEED IN 10 MIN. WILL GIVE BRAINLEST Solve the triangle. B = 36°, a = 41, c = 17

Answers

Answer 1

Answer:

Yes this is a Triangle

36 degrees of any side then 41 would connct to 36 and 17 would connects to 36 and 41! If this is Khan Academy your asking out of its a Yes, it is a Triangle

HOPE IM THE BRANLIESS UwU

Answer 2

Answer:

It is a triangle:

Step-by-step explanation:

b² = a² + c² - 2(a)(c)cos(B)  

b² = 41² + 20² - 2(41)(20)cos(36)

b² = 754.2121292

b = 27.46292281

b = 27.463

A = 41, B = 27.4, C = 17

NEED IN 10 MIN. WILL GIVE BRAINLEST Solve The Triangle. B = 36, A = 41, C = 17

Related Questions

If x3 + ax2 – bx + 10 is divisible by x2 – 3x + 2,
find the values of
1) a-b
2) 2a-b

Answers

Answer: A=2 and B=13
Explanation: The Factor Theorem states that if a is the root of any polynomial p(x) that is if p(a)=0, then (x−a) is the factor of the polynomial p(x).

Let p(x)=x
3
+ax
2
−bx+10 and g(x)=x
2
−3x+2
Factorise g(x)=x
2
−3x+2:
x
2
−3x+2=x
2
−2x−x+2=x(x−2)−1(x−2)=(x−2)(x−1)
Therefore, g(x)=(x−2)(x−1)
It is given that p(x) is divisible by g(x), therefore, by factor theorem p(2)=0 and p(1)=0. Let us first find p(2) and p(1) as follows:
p(1)=1
3
+(a×1
2
)−(b×1)+10=1+(a×1)−b+10=a−b+11
p(2)=2
3
+(a×2
2
)−(b×2)+10=8+(a×4)−2b+10=4a−2b+18
Now equate p(2)=0 and p(1)=0 as shown below:
a−b+11=0
⇒a−b=−11.......(1)
4a−2b+18=0
⇒2(2a−b+9)=0
⇒2a−b+9=0
⇒2a−b=−9.......(2)
Now subtract equation 1 from equation 2:

(2a−a)+(−b+b)=(−9+11)
⇒a=2
Substitute a=2 in equation 1:
2−b=−11
⇒−b=−11−2
⇒−b=−13
⇒b=13
Hence, a=2 and b=13.

Pls help, I don’t know how to fo

Answers

frustum of a cone is: = pi * l(R + r)

(l) = slant height of the frustum.

from 2929.645714 - 506.1257143

= 2423.52

= 2423.5cm

Answer:

from 2929.645714 - 506.1257143

= 2423.52

= 2423.5cm

A box contains 20 oranges and 10 grapes what is the probability of picking a grape from the box?​

Answers

Answer:

[tex]\frac{1}{3}[/tex]

Step-by-step explanation:

First, let's find how many total items there are in the box.

If there are 20 oranges and 10 grapes, then there are [tex]20+10=30[/tex] items in the box.

Now, if there are 10 grapes in this box, we know that the probability of picking a grape is [tex]\frac{10}{30}[/tex] because the total is the denominator and the number of items for that selection is the numerator.

We can simplify this fraction down to [tex]\frac{1}{3}[/tex].

Hope this helped!

Answer:

1/3

Step-by-step explanation:

A
man paid 15600
for a new
car. He
was given a discount of
7%. Find the marked price
of the car​

Answers

hope it helps.I was reading the same chapter

find the perimeter of the quadrant whose radius is 21cm​

Answers

Answer:

75 cm

Step-by-step explanation:

∅=90° , r = 21 cm

Arc length= (2πr∅)/360

=(2π×21×90)/360

=33 cm

Perimeter= arc length + 2(radius)

=33+2(21)

=33 + 42

= 75 cm

3. Solve 2log4y - log4 (5y - 12) = 1/2

Answers

Answer:

y =  4  or y = 6

Step-by-step explanation:

2log4y - log4 (5y - 12) = 1/2

​2log_4(y) - log_4(5y-12) = log_4(2)           apply law of logarithms

log_4(y^2) + log_4(1/(5y-12)) = log_4(/2)    apply law of logarithms

log_4(y^2/(5y-12)) = log_4(2)                     remove logarithm

y^2/(5y-12) = 2                                            cross multiply

y^2 = 10y-24                                                  rearrange and factor

y^2 - 10y + 24 = 0

(y-4)(y-6) = 0

y= 4 or y=6

Solve for x. 3x-91>-87 AND 17x-16>18

Answers

Answer & Step-by-step explanation:

For this problem, we have two inequalities to solve for x.

3x - 91 > -87

17x - 16 > 18

Now that we know what our inequalities are, we will solve them as if we are solving for the value of x.

3x - 91 > -87

Add 91 on both sides.

3x > 4

The solution for the first inequality is 3x > 4

Now let's do the second inequality.

17x - 16 > 18

Add 16 on both sides.

17x > 34

Divide by 17 on both sides.

x > 2

The soultion for the second inequality is x > 2

Answer:

The answer is x>2

Step-by-step explanation:

1. 3x + 6y = 3 and 7x + 3y = 7
ons for bo​

Answers

Answer:

(1,0)

Step-by-step explanation:

3x + 6y = 3

7x + 3y = 7

Multiply the second equation by -2

-2( 7x + 3y = 7)

-14x -6y = -14

Add this to the first equation to eliminate y

3x + 6y = 3

-14x -6y = -14

--------------------

-11 x = -11

Divide by -11

x = 1

Now find y

3x + 6y = 3

3 +6y = 3

Subtract 3 from each side

6y = 0

y =0

Answer:

x = 1

y = 0

Step-by-step explanation:

3x + 6y = 3

7x  + 3y = 7

=> 3y = 7 - 7x

=> y = -7/3x + 7/3

3x + 6(-7/3x + 7/3) = 3

=> 3x - 14x + 14 = 3

=> -11x = -11

=> -x = -11/11

=> -x = -1

=> x = 1

So, 3(1) + 6y = 3

=> 3 + 6y = 3

=> 6y = 0

=> y = 0/6

=> y = 0

So, x = 1

      y = 0

Which of the following is true? Tangent is positive in Quadrant I. Sine is negative in Quadrant II. Cosine is positive in Quadrant III. Sine is positive in Quadrant IV.

Answers

A) Tangent is positive in Quadrant I.

Since sine and cosine are both positive in Quadrant I and tangent is the ratio of sine to cosine, tangent is positive in Quadrant I

Answer:

A

Step-by-step explanation:

I had this question and got it right the user above explains it in detail

A blimp is 1100 meters high in the air and measures the angles of depression to two stadiums to the west of the blimp. If those measurements are 75.2° and 17.9°, how far apart are the two stadiums?

Answers

Answer:

The two stadiums are approximately 3115.1 meters away from each other

Step-by-step explanation:

Since we can construct two right angle triangles between the blimp and the two stadiums as shown in the attached image, then the distance "x" between the two can be find as the difference between the right triangle legs that extend on the ground.

In order to find the size of such legs, one can use the tangent function of the given depression angles as shown below:

[tex]tan(75.2^o)=\frac{1100}{a} \\a=\frac{1100}{tan(75.2^o)}\\a\approx 290.6\,\,meters[/tex]

and for the other one:

[tex]tan(17.9^o)=\frac{1100}{b} \\b=\frac{1100}{tan(17.9^o)}\\b\approx 3405.7\,\,meters[/tex]

The the distance between the stadiums is the difference:

b - a = 3405.7  - 290.6 meters = 3115.1  meters

Raj tested his new flashlight by shining it on his bedroom wall. The beam of light can be described by the equation . How many inches wide is the beam of light on the wall?

Answers

Answer:

12 inches

Step-by-step explanation:

Raj tested his new flashlight by shinning it on his bedroom wall the beam of the light can be described by the equation (x^2-2x) + (y^2-4y) - 31=0. how many inches wide is the beam of light on the wall

Solution

Given:

(x^2-2x) + (y^2-4y) - 31=0

By completing the square

(x^2-2x) + (y^2-4y) - 31=0

(x^2-2x+1-1) + (y^2-4y+4-4)-31=0

(x-1)^2 -1 + (y-2)^2 - 4 - 31=0

(x-1)^2 + (y-2)^2 - 1 - 4 - 31=0

(x-1)^2 + (y-2)^2 - 36=0

(x-1)^2 + (y-2)^2=36

Writing the equation in the form: (x-h)^2+(y-k)^2=r^2

(x-1)^2+(y-2)^2=6^2

From the above, r=6

Where,

r=radius

how wide is the diameter ?

radius=6

Diameter= 2 × radius

=2×6

=12 inches

Answer:

12

Step-by-step explanation:

to graph it just scan the equation on photo math!!

In which direction must the graph of Ax) = x be shifted to produce the graph of g(x) = f(x) - 4?
ОА. up
OB. down
O c. left and down
OD. right and up​

Answers

Answer: B. down

Step-by-step explanation:

Translation rules:

For a function h(x):

h(x+c) is a left-shift by c units.h(x-c) is a right-shift by c units.h(x)+c is a up-shift by c units.h(x)-c is a down-shift by c units.

Here, the graph of f(x) becomes the graph of g(x) =f(x)-4 which is similar to "h(x)-c".

That means , f(x) is shifted 4 units down to become g(x).

So, correct option : B. down

Can someone help me solve parts (a) and (c) please? Thank you!

Answers

a) 4x +6

Add up all the sides to calculate perimeter

Answer:

a) 6x + 6

b) 15 x 24

c) see explanation

Step-by-step explanation:

a) 2x + x + 3 + 2x + x + 3 = 6x + 6

b) 6x + 6 = 78

6x = 72

x = 12

2(12) = 24

(12) + 3 = 15

15 x 24

c) 2x(x + 3) = 2x² + 6x

2(12)² + 6(12) = 288 + 72 = 360

15 x 24 is also 360

To test the belief that sons are taller than their​ fathers, a student randomly selects 13 fathers who have adult male children. She records the height of both the father and son in inches and obtains the following data. Are sons taller than their​ fathers? Use the alphaequals0.10 level of significance.​ Note: A normal probability plot and boxplot of the data indicate that the differences are approximately normally distributed with no outliers.
Height of Father Height of Son
72.4 77.5
70.6 74.1
73.1 75.6
69.9 71.7
69.4 70.5
69.4 69.9
68.1 68.2
68.9 68.2
70.5 69.3
69.4 67.7
69.5 67
67.2 63.7
70.4 65.5
Which conditions must be met by the sample for this​ test? Select all that apply.
A. The sample size is no more than​ 5% of the population size.
B. The differences are normally distributed or the sample size is large.
C. The sample size must be large.
D. The sampling method results in a dependent sample.
E. The sampling method results in an independent sample.
Write the hypotheses for the test. Upper
H 0 ​:
H 1 ​:
Calculate the test statistic. t 0=? ​
(Round to two decimal places as​ needed.)
Calculate the​ P-value. ​P-value=?
​(Round to three decimal places as​ needed.) Should the null hypothesis be​ rejected?
▼ Do not reject or Reject Upper H 0 because the​ P-value is ▼ less than or greater than the level of significance. There ▼ is or is not sufficient evidence to conclude that sons ▼ are the same height or are shorter than or are taller than or are not the same height as their fathers at the 0.10 level of significance. Click to select your answer(s).

Answers

Answer:

1) B. The differences are normally distributed or the sample size is large

C. The  sample size mus be large

E. The sampling method results in an independent sample

2) The null hypothesis H₀:  [tex]\bar x_1[/tex] =  [tex]\bar x_2[/tex]

The alternative hypothesis Hₐ: [tex]\bar x_1[/tex] <  [tex]\bar x_2[/tex]

Test statistic, t = -0.00693

p- value = 0.498

Do not reject Upper H₀ because, the P-value is greater than the level of significance. There is sufficient evidence to conclude that sons are the same height as their fathers  at 0.10 level of significance

Step-by-step explanation:

1) B. The differences are normally distributed or the sample size is large

C. The  sample size mus be large

E. The sampling method results in an independent sample

2) The null hypothesis H₀:  [tex]\bar x_1[/tex] =  [tex]\bar x_2[/tex]

The alternative hypothesis Hₐ: [tex]\bar x_1[/tex] <  [tex]\bar x_2[/tex]

The test statistic for t test is;

[tex]t=\dfrac{(\bar{x}_1-\bar{x}_2)}{\sqrt{\dfrac{s_{1}^{2} }{n_{1}}-\dfrac{s _{2}^{2}}{n_{2}}}}[/tex]

The mean

Height of Father, h₁,  Height of Son h₂

72.4,      77.5

70.6,      74.1

73.1,       75.6

69.9,      71.7

69.4,      70.5

69.4,      69.9

68.1,       68.2

68.9,      68.2

70.5,       69.3

69.4,       67.7

69.5,       67

67.2,       63.7

70.4,       65.5

[tex]\bar x_1[/tex]  = 69.6      

s₁ = 1.58

[tex]\bar x_2[/tex] = 69.9

s₂ = 3.97

n₁ = 13

n₂ = 13

[tex]t=\dfrac{(69.908-69.915)}{\sqrt{\dfrac{3.97^{2}}{13}-\dfrac{1.58^{2} }{13}}}[/tex]

(We reversed the values in the square root of the denominator therefore, the sign reversal)

t = -0.00693

p- value = 0.498 by graphing calculator function

P-value > α Therefore, we do not reject the null hypothesis

Do not reject Upper H₀ because, the P-value is greater than the level of significance. There is sufficient evidence to conclude that sons are the same height as their fathers  at 0.10 lvel of significance

Please answer quickly! A radio telescope has a parabolic surface, as shown below. A parabola opening up with vertex at the origin is graphed on the coordinate plane. The height of the parabola from top to bottom is 1 meter and its width from left to right is 20 meters. If the telescope is 1 m deep and 20 m wide, how far is the focus from the vertex?

Answers

Answer:

Basing on the description, a parabola checking with vertex at origin, the formula with vertex at origin can be used, x^2 = 4py. p is the focus therefore with the dimensions given, we get yourself a 0.25 and this is the distance of the focus to the vertex.

What is the perimeter of this polygon?
A(2, 3)
B(-4, 0)
C(0,-4)
D(4,0)​

Answers

Answer:2,3 hope it help you

Step-by-step explanation:

Answer:

21.627

Step-by-step explanation:

get the distance between all points then add

Factorise the following

Answers

Answer:

4ny²+4n²-4n-8+y⁴-2y²

Find the height of the triangle by applying formulas for the area of a triangle and your knowledge about triangles.
A. 10.5 cm
B. 3.4 cm
C. 8.5 cm
D. 12 cm

Answers

Answer:

12 cm is the right answer pls mark me brainliest

The height of the triangle by applying formulas for the area of a triangle and your knowledge about triangles is 12 cm.

What is Area of Triangle?

The area of a triangle is defined as the total space occupied by the three sides of a triangle in a 2-dimensional plane. The basic formula for the area of a triangle is equal to half the product of its base and height, i.e., A = 1/2 × b × h.

What is Heron's formula?

Heron's formula, formula credited to Heron of Alexandria (c. 62 ce) for finding the area of a triangle in terms of the lengths of its sides. In symbols, if a, b, and c are the lengths of the sides:

Area = √s(s - a)(s - b)(s - c) where s is half the perimeter, or (a + b + c)/2.

Given:

Three sides are: 15cm, 25 cm and 2 cm

Now, Using Heron's formula

semi-perimeter= (25+ 20 + 15)/2

s= 30 cm

Now,

Area of triangle

=√s(s-a)(s-b)(s-c)

=√30* 5 * 10* 15

=√5*2*3*5*2*5*3*5

=5*5*2*3

=150 cm²

Again, area of triangle= 1/2* b* h

150= 1/2* 25* x

12cm= x

Learn more about Area of Triangle here:

https://brainly.com/question/9817285

#SPJ2

Is 7 a solution of 5x - 3 = 12?

Answers

Answer:

no

Step-by-step explanation:

5x - 3 = 12

5x=12+3

x=15/5=3

x=3

substitute x with 7 to find if it is a solution :

5(7)-3=12

35-3 ≠12 ( so no 7 is not a solution)

simplify the equation. (5xE2 - 3x) - (5xE2 - 3x+1)

Answers

Answer:

[tex]\huge \boxed{\mathrm{-1}}[/tex]

Step-by-step explanation:

[tex](5xe^2 - 3x) - (5xe^2 - 3x+1)[/tex]

Distribute negative sign.

[tex]5xe^2 - 3x- 5xe^2 +3x-1[/tex]

Combine like terms.

[tex]0xe^2 +0x-1[/tex]

[tex]0-1=-1[/tex]

Latanya buys 5 yard of blue fabric and 8 yards of green fabric. the blue fabric cost $2 dollars more than the green fabric.she pays a total of $ 62. what would be the combined cost of 1 yard of blue fabric and one yard of green fabric?

Answers

Answer: $10

Step-by-step explanation:

let x = the price of green fabric, then x+2 = blue fabric price

8x+5(x+2)=62

8x+5x+10=62

    13x+10=62

          13x=52

              x=4

price of green fabric=$4

price of blue fabric=$6

4+6=$10

One type of fabric costs $31.25 for 5 square yards. Another type of fabric costs $71.50 for 11
square yards. Is the relationship between the number of square yards and the cost
proportional between the two types of fabric?

Answers

Answer:

as ratio of two type of fabric is different .

hence, the relationship between the number of square yards and the cost

is not proportional between the two types of fabric

Step-by-step explanation:

For a relation to be proportional

a:b = c:d

in other form

a/b = c/d

______________________________________________

Ratio for first type of fabric

cost of fabric/ area of fabric = 31.25/5 = 6.25

Ratio for other type of fabric

cost of fabric/ area of fabric = 71.50/11 = 6.5

as ratio of two type of fabric is different .

hence, the relationship between the number of square yards and the cost

is not proportional between the two types of fabric

10. Write a word problem for this equation:
n ($25) = $125

Answers

Answer:

The word problem is "How many $25 are there in $125?"

Step-by-step explanation:

Given

[tex]n(\$25) = \$125[/tex]

Required

Write a word problem for the expression

We start by solving the given equation

[tex]n(\$25) = \$125[/tex]

Divide both sides by $25

[tex]\frac{n(\$25)}{\$25} = \frac{\$125}{\$25}[/tex]

[tex]n = \frac{\$125}{\$25}[/tex]

[tex]n = 5[/tex]

This implies that there are 5, $25 in $125

Hence; The word problem is "How many $25 are there in $125?"

If the measure of angle 4 is (11 x) degrees and angle 3 is (4 x) degrees, what is the measure of angle 3 in degrees?

Answers

Answer:

is it 2

Step-by-step explanation:

Georgina has $5.60 in quarters and dimes. If Georgina has 14 more dimes than quarters, how many quarters does she have?
[I don't want the answer i just want someone to show me how the problem is set up because its confusing me...Thanks]

Answers

Answer:

She has 12 quarters.

Step-by-step explanation:

You have two unknowns, the number of quarters and the number of dimes. You need one equation per unknown, so you need two equations. One equation deals with the number of coins. The other equation deals with the values of the coins.

The first step is to choose variables. We can go with x and y, but I prefer to choose q and d, so I remember more easily what they represent.

Let q = number of quarters.

Let d = number of dimes.

Let's deal with the numbers of coins first.

"Georgina has 14 more dimes than quarters"

She has q number of quarters, but the number of dimes is 14 more than the number of quarters. d is 14 more than q. Our first equation is

d = q + 14

Now we deal with the values of the coins.

We don't know yet the actual numbers of dimes and quarters, so we use d and q to represent those numbers.

One dime is worth $0.10; d number of dimes is worth 0.1d.

A quarter is worth $0.25; q number of quarters is worth 0.25q.

The total value of the dimes and quarters is the sum of the values of the two sets of coins:

0.1d + 0.25q

We are told "Georgina has $5.60 in quarters and dimes."

This gives us our second equation.

0.1d + 0.25q = 5.6

Now we have a set of two equations in two variables:

d = q + 14

0.1d + 0.25q = 5.6

There are several methods for solving a system of equations. Since the first equation is already solved for a variable, d, we can use the substitution method.

We rewrite the second equation, but we substitute q + 14 for d in the second equation.

Second equation:

0.1d + 0.25q = 5.6

Substitute q + 14 for d:

0.1(q + 14) + 0.25q = 5.6

Distribute on the left side:

0.1q + 1.4 + 0.25q = 5.6

Combine like terms on the left side:

0.35q + 1.4 = 5.6

Subtract 1.4 from both sides:

0.35q = 4.2

Divide both sides by 0.35:

q = 12

There are 12 quarters.

(You are asked only the number of quarters, so you can stop here. I will continue to find also the number of dimes for 2 reasons. 1) You see how it's done, so it will help with other problems. 2) By finding the numbers of dimes and quarters, then we can check if our solution is correct, which I will do below at the end.)

Now we use the first equation, d = q + 14. We substitute 12 for q and solve for d.

d = q + 14

d = 12 + 14

d = 26

There are 26 dimes.

Check:

We check the numbers of coins:

26 - 12 = 14 The number of dimes is indeed 14 more than the number of quarters.

We check the values of the coins:

0.1(26) + 0.25(12) = 2.6 + 3 = 5.6 The value of the coins is indeed $5.60.

Our answer is correct.

I need help ASAP!!

Can someone explain this? And answer it? I am so confused!!

Answers

Answer:

Step-by-step explanation: hope this helps

Help the question is there

Answers

Answer:

y = 7 when x = -5

Step-by-step explanation:

First go to x = -5

Then go up to where you meet the green line

The y value is 7

y = 7 when x = -5

what is the value of a in the functions equation?

Answers

Answer:

y= -2x^2 + 12x -14

Step-by-step explanation:

when I put the points into a graphing calculator this is what I got so I think the answer is -2

Answer:

Step-by-step explanation:

It is -2

Which of the following is an irrational number?
5 / 4
√5 / 7
1/ 8
3 / 5

Answers

Answer:  [tex]\sqrt{5} /7[/tex]

Step-by-step explanation:

5/4 is not an irrational number because it is already in a fraction the same as 1/8 and 3/5.

The square root of 5 is not rational because it cannot be converted to a fraction or in other words is not a perfect square.

Answer:√5 / 7

Step-by-step explanation:

If you make $3.80 an hour plus tips, what is your paycheck for the week if you worked 40 hours and made $250.00 dollars in tips?

Answers

Answer:

$402

Step-by-step explanation:

Hello!

If you made 3.80 an hour and worked 40 we can multiply these to find the total amount you earned.

3.80 * 40 = 152

You also made 250 in tips so we add that to the total

152+250 = 402

The answer is $402

Hope this helps!

I would make $402.00 by the end of the week
Other Questions
Match each statement with its corresponding value for the system below:y = -2(3)x and y = 9x - 21.The number of points of intersection.2.The x-coordinate of the solution.3.The y-coordinate of the solution. A group of coordinately regulated structural genes with related metabolic functions, plus the promoter and operator sites that control their transcription, is called a/an ___________. Eddies saving account has a balance of $140. He deposits $17 for each of the next five weeks. He withdraws $11 in the final week. He wants to use his savings to buy a game system for $210. Do he have enough money to buy the game system? A company decides not to pay dividends to stockholders, but the company is requested to pay interest to debt holders. What does this mean about the performance of the company? Go! That was the only thoght going threw Henrys mind. He had to get home and warn his sisster. What would happen if he did not get there in time. That was a though Henry could not waste on. He pushed his feet faster and faster. His sister was who he must get to. What is the issue with this creative writing?A. It needs to be proofreadB. It lacks a variety of sentence structure.C. It is full of clichsD. There are no issues The data show the number of hours of television watched per day by a sample of 28 people. Use technology to answer parts (a) and (b) below. 1 1 2 8 8 4 8 7 8 3 1 2 8 2 4 7 4 0 5 7 7 8 9 3 6 2 2 7 a. Find the data set's first, second, and third quartiles. Upper Q 1 equals nothing Upper Q 2 equals nothing Upper Q 3 equals nothing Which strategy is considered a timeout? captive company rebirth pause/proceed-with-caution contraction concentration EarthShapers Inc., a manufacturer of heavy construction equipments based in U.S.A., maintains large sales and support operations overseas. Before sending new employees to its operations in Japan, it requires the employees to take courses in Japanese history and culture so that they can adjust more easily to living in Japan. This training focuses on the _____ component of international competence training. Je m'appelle Isabelle et j'habite Londres mais je suis canadienne. L't je vais au Canada rendre visite ma famille canadienne. J'ai mes grands-parents et mes oncles et tantes au Canada. Mes grands-parehabitent Qubec et mes oncles et tantes Montral. Ma grand-mre n'aime pas voyager en avion, alors ils ne voyagent pas en Angleterre. Avec mes cousines nous allons en ville faire les magasins et mangerestaurant. J'adore faire les achats Montral. Souvent nous allons jouer au tennis le samedi matin et aprs nous faisons un pique-nique dans le parc. Je passe le mois de juillet l-bas et mes cousines vont erAngleterre au mois d'aot. J'aime beaucoup ma famille.Qu'est-ce qu'Isabelle aime faire en ville? (1 point)jouer au tennisfaire un pique-niqueCvoyager en avion0aller dans les magasins What occurs at the end of a females monthly cycle? PLEASE HELP ME!!!!!!!!!!!! PLEASE HELP ME!!!!!!!!!!!! PLEASE HELP ME!!!!!!!!!!!! PLEASE HELP ME!!!!!!!!!!!! PLEASE HELP ME!!!!!!!!!!!! PLEASE HELP ME!!!!!!!!!!!! PLEASE HELP ME!!!!!!!!!!!! PLEASE HELP ME!!!!!!!!!!!! Please help! Algebra 1. Water is pumped with a 120 kPa compressor entering the lower pipe (1) and flows upward at a speed of 1 m/s. Acceleration due to gravity is 10 m/s and water density is1000 kg/m-3. What is the water pressure on the upper pipe (II). Century Roofing is thinking of opening a new warehouse, and the key data are shown below. The company owns the building that would be used, and it could sell it for $100,000 after taxes if it decides not to open the new warehouse. The equipment for the project would be depreciated by the straight-line method over the project's 3-year life, after which it would be worth nothing and thus it would have a zero salvage value. No new working capital would be required, and revenues and other operating costs would be constant over the project's 3-year life. What is the project's NPV? (Hint: Cash flows are constant in Years 1-3.) Dog Up! Franks is looking at a new sausage system with an initial cost of $445,000 that will last for five years. The fixed asset will qualify for 100 percent bonus depreciation in the first year, at the end of which the sausage system can be scrapped for $53,000. The sausage system will save the firm $139,000 per year in pretax operating costs, and the system requires an initial investment in net working capital of $25,000. If the tax rate is 23 percent and the discount rate is 11 percent, what is the NPV of this project? (Do not round intermediate calculations and round your answer to 2 decimal places, e.g., 32.16.) in judiths cofers "gravity," how do elenitas mother and father conlict? a 50g sample of radioactive iodine-131 has a half-life of 8.0 days. After 32 days, how much is left? Which of the following would be a thesis statement? aIn this essay, I will show that prospective pet owners should adopt an animal from a shelter. bPuppies purchased from puppy mills often carry fatal illness such a heartworm and parvovirus. cProspective pet owners should consider adopting an animal from a a shelter in order to ensure that their pet is healthy. The world on the turtles back Answer the following questions properly (complete sentences and direct citations/quotations where necessary) at some point, the audience of this story probably decided the world on the turtles back is a metaphorical, or figurative, and not literal. When do you think this occurred? Do you think it changed the meaning of the story? Why or why not? The one-sample z test is: a. a hypothesis test b. used to test hypotheses c. concerning a single population with a known variance d. concerning at least one population e. concerning the variance in a population d. all of the above